Exercices

Exercice 1   En examinant la limite du terme général, montrer que les séries suivantes divergent.

$\displaystyle \sum \sin(n)
\;;\quad
\sum (1+(-1)^n\cos(1/n))
\;;\quad
\sum \left(\frac{n}{{n+1}}\right)^n
\;;\quad
\sum \frac{(-1)^n}{1+n^{-1}}
$

Exercice 2   Utiliser le théorème de comparaison ou un équivalent, pour démontrer que
  1. les séries suivantes convergent

    $\displaystyle \sum \frac{1-\cos(\frac{1}{n\ln(n)})}
{\sin(\frac{1}{n})}
\;;\quad
\sum \sqrt{n+2} \;\sin\left(\frac{3}{(n+1)^2}\right)
\;;\quad
$

    $\displaystyle \sum \left(\frac{1}{n}+\ln \left(\frac{n-1}{n}\right)\right)
\;;\quad
\sum \vert\sin(\pi\sqrt{n^4+1})\vert^{\frac{3}{4}}
\;;\quad
$

    $\displaystyle \sum \frac{n^2}{2^n+n}
\;;\quad
\sum \mathrm{e}^{n^\frac{1}{3}-\sqrt{n}}
\;;\quad
$

    $\displaystyle \sum \left(1-\cos \left(\frac{n-1}{n}\right)\right)
\;;\quad
\sum \arg\!\cosh \frac{n+1}{n}
\;.\quad
$

  2. les séries suivantes divergent

    $\displaystyle \sum \ln\left(\frac{n+1}{n}\right)
\;;\quad
\sum \sqrt{n^2+2n}\; \sin\left(\frac{3}{(n+1)^2}\right)
\;;\quad
$

    $\displaystyle \sum n^{-(1+\frac{1}{n})}
\;;\quad
\sum \frac{1}{n+(-1)^n \sqrt{n}}
\;;\quad
$

    $\displaystyle \sum \sqrt{n^2 +n+1}-\sqrt{n^2+n-1}
\;;\quad
\sum (n^3 +1)^{\frac{1}{3}}-\sqrt{n^2+1}
\;.\quad
$

Exercice 3   Utiliser le critère de Cauchy pour déterminer la nature des séries suivantes.

$\displaystyle \sum \frac{n^{\ln n}}{(\ln n)^n}
\;;\quad
\sum \left(\frac{n+3}{2n+1}\right)^n
\;;\quad
$

$\displaystyle \sum \left(\frac{n+3}{2n+1}\right)^{n(-1)^n}
\;;\quad
\sum \left(\frac{n}{2n+1}\right)^{n^2}
\;.
$

Exercice 4   Utiliser le critère de d'Alembert pour déterminer la nature des séries suivantes.

$\displaystyle \sum \frac{n!}{n^n}
\;;\quad
\sum \frac{(\ln n)^n}{n!}
\;;\quad
\sum \frac{(3n)!}{9(n!)^3}
$

$\displaystyle \sum \frac{n!}{n^{\alpha n}}
\;;\quad
\sum \frac{n^\alpha (\ln n)^n}{n!}
\;;\quad
\sum \frac{2^n}{n^2\sin^{2n}(\alpha)}\;.
$

(discuter selon la valeur du réel $ \alpha$).

Exercice 5   Déterminer la nature des séries suivantes

$\displaystyle \sum n\mathrm{e}^{-n}\;;\quad
\sum n\mathrm{e}^{-\sqrt{n}}\;;\qua...
...rac{\mathrm{e}^n}{n^5+1}\;;\quad
\sum \frac{\mathrm{e}^{-n}}{4+\sin n}\;;\quad
$

$\displaystyle \sum \frac{2^n+n^2}{3^nn^2+1}\;;\quad
\sum \frac{\mathrm{e}^{\fra...
...ac{1}{n}}-1}{\sqrt{n+1}}\;;\quad
\sum n\ln\left(1+{\frac{1}{n}}\right)\;;\quad
$

$\displaystyle \sum \frac{1-n\ln\left(1+{\frac{1}{n}}\right)}{\sqrt{n+1}}\;;\qua...
...1}{\sqrt{n}}\right)^n\;;\quad
\sum \left(1-\frac{1}{\sqrt{n}}\right)^n\;;\quad
$

$\displaystyle \sum \frac{n!}{n^n}\;;\quad
\sum \frac{\sin n}{n^2+\cos^2 n}\;;\q...
...{(n+1)\pi }\frac{\sin x}{x^2}  \mathrm{d}x\;;\quad
\sum (\ln n)^{-n} \;;\quad
$

$\displaystyle \sum \left(\frac{n+3}{2n+1}\right)^{n\ln n}\;;\quad
\sum (\mathr...
...frac{1}{n}})-\cos \frac{1}{ n^{1/2}}\;;\quad
\sum \frac{(n!)^2}{(2n)!}\;;\quad
$

$\displaystyle \sum \frac{1}{n.n^{3/n}}\;;\quad
\sum \frac{\sqrt{n(n-1)}}{n^3 -2...
...3\ln n}\;;\quad
\sum \frac{\sqrt{n(n-\cos(n))}}{\sqrt{n^4 -2n^3}+3\sin(n)}
\;.
$

Exercice 6   On considère une suite $ (u_n)$ définie par $ u_0\in\mathbb{R}^+$ et pour tout $ n\in\mathbb{N}$ par l'une des formules suivantes.
$ \bullet$
$ u_{n+1}=\frac{\sin(u_n)}{n+1}$
$ \bullet$
$ u_{n+1}=\frac{\ln(1+u_n)}{2}$
$ \bullet$
$ u_{n+1}=1-\cos(u_n)$
  1. Montrer que $ (u_n)$ tend vers 0.
  2. Étudier la limite du rapport $ u_{n+1}/u_n$.
  3. En déduire que la série $ \sum
u_n$ est convergente.

Exercice 7   Soient $ a$ et $ b$ deux réels. Pour tout $ n\in\mathbb{N}$, on pose

$\displaystyle u_n=\sqrt{n}+a\sqrt{n+1}+b\sqrt{n+2}\;.$

  1. Vérifier que la suite $ (u_n)$ tend vers 0 si et seulement si $ a+b=-1$.
  2. Déterminer $ a$ et $ b$ pour que la série $ \sum
u_n$ soit convergente.

Exercice 8   Soit $ \sum
u_n$ une série à termes positifs ou nuls, convergente.
  1. Montrer que pour tout $ \alpha>1$, la série $ \sum u_n^\alpha$ converge.
  2. Montrer que les séries $ \sum \sin(u_n)$ et $ \sum \arctan(u_n)$ convergent.
  3. Soit $ f$ une application de $ \mathbb{R}^+$ dans $ \mathbb{R}^+$ telle que $ f(0)=0$, admettant une dérivée à droite en 0. Montrer que la série $ \sum f(u_n)$ converge.

Exercice 9   Soit $ u_n$ une suite à termes réels positifs ou nuls. Montrer que les séries de termes généraux $ u_n$, $ \frac{u_n}{1+u_n}$, $ \ln (1+u_n)$ et $ \int_0^{u_n}\frac{\mathrm{d}x}{1+x^3}$ sont de même nature (convergentes ou divergentes).

Exercice 10   Soient $ \sum
u_n$ et $ \sum v_n$ deux séries à termes positifs ou nuls, convergentes.
  1. Montrer que les séries $ \sum \min\{ u_n,v_n\}$ et $ \sum \max\{
u_n,v_n\}$ convergent.
  2. Si $ a$ et $ b$ sont deux réels tels que $ 0<a<b$, alors $ 0<a<2ab/(a+b)<\sqrt{ab}<b$ ($ 2ab/(a+b)$ et $ \sqrt{ab}$ sont respectivement la moyenne harmonique et la moyenne géométrique de $ a$ et $ b$). Montrer que $ \sum\sqrt{u_nv_n}$ et $ \sum \frac{u_nv_n}{u_n+v_n}$ convergent.

Exercice 11   Soit $ \sum u_{n}$ et $ \sum v_{n}$ deux séries réelles convergentes et $ \sum w_{n}$ une série réelle telle que pour tout $ n\in\mathbb{N}$,

$\displaystyle u_{n}\leqslant w_{n}\leqslant v_{n}\;.
$

  1. Montrer que pour tout $ n\in\mathbb{N}$,

    $\displaystyle 0\leqslant w_{n}-u_{n}\leqslant v_{n}-u_{n}
$

  2. En déduire que $ \sum w_{n}$ converge.

Exercice 12   Soit $ f$ la fonction définie sur $ [2,\infty[$ par $ f(x)=\frac{1}{x\ln x}$. Soit pour tout $ n\geqslant 2$, $ u_n=f(n)$ et $ s_n=\sum_{k=2}^n u_k$.
  1. Vérifier que $ F : x\mapsto \ln(\ln(x))$ est une primitive de $ f$. En déduire les inégalités :

    $\displaystyle s_n\geqslant F(n+1)-F(2)$   et$\displaystyle \quad
s_n-u_2\leqslant F(n)-F(2)
$

  2. Déduire de la question précédente que $ s_n$ est équivalent à $ \ln(\ln(n)$ quand $ n$ tend vers l'infini.
  3. On pose $ a_n=s_n-\ln(\ln n)$. Montrer en utilisant des développements limités que la série de terme général $ (a_n-a_{n-1})$ converge. En déduire que $ S_n-\ln(\ln n)$ est borné.
  4. Quelle est la plus petite valeur de $ n$ telle que $ s_n>10^3$ ?

Exercice 13   Pour tout $ n\geqslant 1$, on note $ u_n=1/\sqrt{n}$ et

$\displaystyle s_n=\sum_{k=1}^n u_n
$

  1. Montrer que pour tout $ n\geqslant 2$,

    $\displaystyle S_n-1\leqslant 2(\sqrt{n}-1)\leqslant S_{n-1}
$

  2. En déduire que $ S_n$ est équivalent à $ 2\sqrt{n}$ quand $ n$ tend vers l'infini.
  3. Quelle est la plus petite valeur de $ n$ telle que $ s_n>10^{-5}$ ?

Exercice 14   En utilisant la comparaison avec une intégrale, démontrer les résultats suivants.

   Si $\displaystyle \gamma\leqslant 1\quad \sum_{n=3}^{+\infty} n^{-1}
(\ln(n))^{-1}(\ln(\ln(n)))^{-\gamma}
\;$ diverge.

   Si $\displaystyle \gamma > 1\quad \sum_{n=3}^{+\infty} n^{-1}
(\ln(n))^{-1}(\ln(\ln(n)))^{-\gamma}
\;$ converge.

Exercice 15   Soit $ u_n$ une suite à termes réels (quelconques).
  1. Donner un exemple tel que $ \sum
u_n$ converge et $ \sum u_n^2$ diverge. On suppose dans les questions suivantes que $ \sum
u_n$ et $ \sum u_n^2$ convergent.
  2. Montrer pour tout $ k\in \mathbb{N}^*$, $ \sum u_n^k$ converge.
  3. Soit $ f$ une application de $ \mathbb{R}$ dans $ \mathbb{R}$ deux fois continûment dérivable, telle que $ f(0)=0$. Montrer que $ \sum f(u_n)$ converge.

Exercice 16   Soit $ n$ un entier strictement positif. On considére l'équation $ x^n +nx+1=0$
  1. Montrer que cette équation admet une unique solution dans $ \mathbb{R}+$. On la note $ u_n$.
  2. Montrer que la suite $ (u_n)$ tend vers 0.
  3. Montrer que la série $ \sum
u_n$ diverge.

Exercice 17   Montrer que les séries suivantes convergent, mais ne sont pas absolument convergentes

$\displaystyle \sum \frac{\sin(n)}{\sqrt{n^2+n}}
\;;\quad
\sum \frac{\sin^3(n)}{\sqrt{n+2}}
\;;\quad
\sum \frac{\cos(n)}{\ln(n)}
\;;\quad
$

$\displaystyle \sum \frac{\sin^5(n)}{\ln(\ln(n))}
\;;\quad
\sum \frac{(-1)^n\cos(n)}{\sqrt n}
\;;\quad
\sum \frac{\sin(3n)\cos(n)}{\sqrt{n}}
$

Exercice 18   Déterminer la nature des séries suivantes.

$\displaystyle \sum \frac{(-1)^n}{ \ln n}\;;\quad
\sum \frac{(-1)^n}{ \sqrt{n}}\...
...\frac{(-1)^n}{ \sqrt{n}}\right)\;;\quad
\sum \frac{(-1)^n}{ n +(-1)^n}\;;\quad
$

$\displaystyle \sum \frac{(-1)^n}{ \sqrt{n} +(-1)^n}\;;\quad
\sum \frac{(-1)^n}{ \sqrt{n+(-1)^n} }\;;\quad
$

$\displaystyle \sum \int_{n\pi }^{(n+1)\pi }\frac{\sin(x)}{x}  \mathrm{d}x\;;\quad
\sum \int_{n\pi }^{(n+1)\pi }\frac{\sin^2(x)}{x}  \mathrm{d}x\;.
$

Exercice 19   Soit $ k$ un entier.
  1. Montrer que $ \cos^{2k}(x)$ est une combinaison linéaire de $ 1,
\cos(2x),\ldots, \cos(2kx)$. En déduire que la série $ \sum \cos^{2k}(n)/n$ est divergente.
  2. Montrer que $ \cos^{2k+1}(x)$ est une combinaison linéaire de $ \cos(x), \ldots, \cos((2k+1)x)$. En déduire que la série $ \sum \cos^{2k+1}(n)/n$ est convergente.

Exercice 20   Soit $ \sum
u_n$ une série convergente à termes complexes. Montrer que la série $ \sum \frac{u_n}{n}$ converge.

Exercice 21   On considère la série $ \sum
u_n$, où

$\displaystyle u_n = \frac{\sin(n)}{\sqrt{n}+ \cos(n)}
$

  1. Vérifier que la suite $ (1/(\sqrt{n}+\cos(n))$ n'est pas décroissante.
  2. Montrer que la suite $ (1/(n-\cos^2(n))$ est décroissante.
  3. Montrer que la suite $ (\sqrt{n}/(n-\cos^2(n))$ est décroissante.
  4. Vérifier que

    $\displaystyle u_n = \frac{\sin(n)\sqrt{n}}{n-\cos^2(n)}-\frac{\sin(n)\cos(n)}{n-\cos^2(n)}\;.
$

    En déduire que $ \sum
u_n$ converge.

Exercice 22   Soient $ a$ et $ b$ deux réels. On considére la série $ \sum
u_n$ avec $ u_n=\frac{a^n}{n+b^n}$.
  1. On suppose $ b\leq 1$. Pour quelles valeurs de $ a$ la série est-elle absolument convergente ?
  2. Même question pour $ b>1$.
  3. On suppose $ a=-1$. Pour quelles valeurs de $ b$ la série est-elle convergente ?
  4. Représenter dans le plan les points de coordonnées $ (a,b)$ tels que la série est absolument convergente, convergente, divergente.

Exercice 23   On considère la série de terme général $ u_n$, où

$\displaystyle u_n = \frac{1}{n(n^2-1)}\;.
$

  1. Écrire la décomposition en éléments simples de la fraction rationnelle $ \frac{1}{X(X^2-1)}$.
  2. En déduire une expression explicite en fonction de $ n$ de

    $\displaystyle \sum_{k=2}^n \frac{1}{k(k^2-1)}
$

  3. Déduire de ce qui précède la convergence de la série $ \sum
u_n$ et la valeur de la somme

    $\displaystyle s=\sum_{k=2}^{+\infty}u_k\;.
$

Exercice 24   On considère la série de terme général :

$\displaystyle u_n=\ln\left(1-\frac{1}{n^2}\right)=
\ln\left(\frac{n^2-1}{n^2}\right)$

  1. Démontrer que cette série converge.
  2. Donner une expression explicite de $ s_n=\displaystyle{\sum_{k=2}^{n}u_k}$.
  3. En déduire la valeur de la somme $ s=\displaystyle{\sum_{k=2}^{+\infty}u_k}$.

Exercice 25   Montrer que les séries suivantes convergent et calculer leurs sommes :

$\displaystyle \sum_{n=3}^{+\infty} 3^{-n+2}+2^{-n+3}\;;\quad
\sum_{n=1}^{+\infty} (n-2)3^{-n}+(n-3)2^{-n}\;;\quad
$

$\displaystyle \sum_{n=1}^{+\infty} (n^2-2n)3^{-n}+(n^2-3n)2^{-n}\;;\quad
\sum_{n=0}^{+\infty} (n^2-2n+3)3^{-n}+(n^2-3n+2)2^{-n}\;;\quad
$

$\displaystyle \sum_{n=1}^{+\infty} \frac{n^2+2n}{n!}\;;\quad
\sum_{n=1}^{+\inft...
...c{n^3+2n^2+3n}{n!}\;;\quad
\sum_{n=1}^{+\infty} \frac{n^3+2n+3n+1}{n!}\;;\quad
$

$\displaystyle \sum_{n=2}^{+\infty} \frac{2n+1}{{n^3-n^2}}
\;;\quad
\sum_{n=2}^...
...c{2n-1}{n^3-4n}
\;;\quad
\sum_{n=3}^{+\infty} \frac{4n+2}{(n^2-1)(n^2-2n)}
\;.
$

Exercice 26   Si $ \sum
u_n$ est une série convergente à termes strictement positifs, on note $ r_n$ son reste d'ordre $ n$ :

$\displaystyle r_n=\sum_{k=n+1}^{+\infty} u_k\;.
$

On suppose qu'il existe $ k\in]0,1[$ et $ n_0\in\mathbb{N}$ tels que $ \frac{u_{n+1}}{u_n}\leqslant k$ pour tout $ n\geqslant
n_0$. Montrer que pour tout $ n\geqslant
n_0$, $ 0<r_n\leqslant \frac{1}{{1-k}}u_{n+1}$.

Exercice 27   Soit $ (a_n)_{n\in\mathbb{N}}$ une suite de réels. On dit que le produit infini $ \prod a_n$ converge, s'il existe $ \pi\in \mathbb{R}^*$ tel que la suite de terme général

$\displaystyle \prod_{k=0}^n a_k$ converge vers $\displaystyle \pi\;.
$

  1. Montrer que si le produit infini $ \prod a_n$ converge, alors $ \forall n ,\; a_n\neq 0$ et

    $\displaystyle \lim_{n\to +\infty} a_n = 1\;.
$

    On pose désormais $ a_n = 1+u_n$.
  2. On suppose que $ \forall n\in \mathbb{N} ,\; a_n>1$. Montrer que le produit $ \prod a_n$ converge si et seulement si la série $ \sum
u_n$ converge.
  3. On suppose que $ \forall n\in \mathbb{N} ,\; a_n\in ]0,1]$. Montrer que le produit $ \prod a_n$ converge si et seulement si la série $ \sum
u_n$ converge.
  4. On suppose que $ \forall n\in \mathbb{N} ,\; a_n\in ]0,+\infty[$. Montrer que si la série $ \sum
u_n$ converge absolument, alors le produit $ \prod a_n$ converge.
  5. Montrer que pour tout $ n\geqslant 2$,

    $\displaystyle \prod_{k=2}^n \left(1-\frac{1}{k^2}\right) = \frac{n+1}{2n}\;.
$

    En déduire $ \displaystyle{\prod_{n=2}^{+\infty}
\left(1-\frac{1}{n^2}\right)}$.
  6. Montrer que

    $\displaystyle \prod_{n=1}^{+\infty} \left(1 +\frac{(-1)^n}{n}\right) = 1
$

    On pourra calculer $ (1+\frac{1}{2n})(1-\frac{1}{2n+1})$.
  7. Montrer que le produit infini

    $\displaystyle \prod \left\vert 1+\frac{\mathrm{i}}{n}\right\vert$    converge.

    On pourra appliquer le résultat de la question 2.
  8. Montrer que le produit infini

    $\displaystyle \prod \left(1+\frac{\mathrm{i}}{n}\right)$    diverge.

    On pourra écrire

    $\displaystyle \left(1+\frac{\mathrm{i}}{n}\right)
=\left\vert 1+\frac{\mathrm{i}}{n}\right\vert \mathrm{e}^{\mathrm{i}\theta_n}$    avec $\displaystyle \theta_n = \arctan\left(\frac{1}{n}\right)\;.
$

Exercice 28   Soit $ (u_n)_{n\in\mathbb{N}}$ une suite de réels, telle que la série $ \sum
u_n$ soit convergente mais non absolument convergente. On note $ (p_n)$ la suite des termes positifs et $ (m_n)$ la suite des termes négatifs.

$\displaystyle p_n = \left\{\begin{array}{ll}
u_n&\mbox{si }u_n> 0\\
0&\mbox{sinon}
\end{array}\right.$   et$\displaystyle \quad
m_n = \left\{\begin{array}{ll}
u_n&\mbox{si }u_n< 0\\
0&\mbox{sinon}
\end{array}\right.
$

  1. Montrer que les séries $ \sum p_n$ et $ \sum m_n$ sont divergentes.
  2. Soit $ a$ un réel quelconque. Construire une bijection $ \sigma_a$ de $ \mathbb{N}$ dans $ \mathbb{N}$ telle que

    $\displaystyle \lim_{n\to+\infty} \sum_{i=1}^n u_{\sigma_a(i)} = a\;.
$

    Indication : supposant $ a>0$, prendre les premiers termes positifs dont la somme dépasse $ a$, puis ajouter des termes négatifs jusqu'à ce que la somme repasse en-dessous de $ a$, puis itérer.
  3. Construire une bijection $ \sigma_+$ de $ \mathbb{N}$ dans $ \mathbb{N}$ telle que

    $\displaystyle \lim_{n\to+\infty} \sum_{i=1}^n u_{\sigma_+(i)} = +\infty\;.
$

  4. Construire une bijection $ \sigma_-$ de $ \mathbb{N}$ dans $ \mathbb{N}$ telle que

    $\displaystyle \lim_{n\to+\infty} \sum_{i=1}^n u_{\sigma_-(i)} = -\infty\;.
$

Exercice 29   Soit $ (u_n)_{n\in\mathbb{N}}$ une suite de réels tels que la série $ \sum
u_n$ soit absolument convergente. Soit $ \sigma$ une bijection de $ \mathbb{N}$ dans $ \mathbb{N}$.
  1. Montrer que la série $ \sum u_{\sigma(n)}$ est absolument convergente.
  2. Montrer que

    $\displaystyle \sum_{n=0}^{+\infty} u_{\sigma(n)} = \sum_{m=0}^{+\infty} u_m\;.
$


         © UJF Grenoble, 2011                              Mentions légales